Tài liệu bồi dưỡng HSG nguyễn quốc thái

199 112 0
Tài liệu bồi dưỡng HSG nguyễn quốc thái

Đang tải... (xem toàn văn)

Tài liệu hạn chế xem trước, để xem đầy đủ mời bạn chọn Tải xuống

Thông tin tài liệu

1 Bất đẳng thức Bài 1 Cho x,y số thực dương cho 2x + y 2y + x khác Tìm giá trị nhỏ biểu ( x  y )(4 x  y ) (2 y  x )(4 y  x ) P   3( x  y ) (2 x  y  2) ( x  y  2) thức Cho a , b, c, > cho a + b + c = Chứng minh a b c   � b (ca  1) c ( ab  1) a (bc  1) (1  abc)( ab  bc  ca ) (THPT chuyên KHTN - ĐH KHTN, ĐHQG Hà Nội) Lời giải Ta chứng minh : (2 x  y )(4 x  y ) �2 x  y  (2 x  y  2) � (2 xy  x  y  2) �0 (đúng) Chứng minh tương tự ta được: P �1 Vậy GTNN P -1 x y x y  65  65 2 �1 1 � �1 1 �   � � �  � c a b c� a b c� � a � � � � 1 1 1 cyc a (bc  1) cyc b   abc   3 c a b c a b c (ab  bc  ca)  2 2 Theo BĐT Cauchy-Schwartz abc(ab  bc  ca )  3a b c Một điều ln x  y �0 x  27 y �0 Vậy BĐT chứng minh, Dấu xảy a = b = c = Đặt ab + bc + ca = x , abc = y BĐT ban đầu ta chứng minh x2 � � x  x y �9 xy  27 y 2 xy  y x(1  y ) � x( x  y)  y ( x  27 y) �0 Bài k k k 3 Tìm số nguyên dương k nhỏ cho bất đẳng thức x y z ( x  y  z ) �3 với số thực dương x , y , z thỏa mãn điều kiện x + y + x = (Trường Phổ thông Năng khiếu, ĐHQG Tp.HCM) Lời giải Lời giải sau trích từ trang nangkhieutoan.com Dễ dàng tìm số để BĐT không với k = k =2 Nhận xét BĐT với k = BĐT với k > x k y k z k ( x3  y  z )  x y z ( x  y  z ).x k 3 y k 3 z k 3 �3 Điều gợi ý cho ta chứng minh k = số nhỏ cần tìm, cách chứng minh x y z ( x3  y  z ) �3 (1.1) Thật vậy, giả sử z số nhỏ ba số x , y , z suy z �1 Ta có x  y  ( x  y)3  xy ( x  y )  (3  z )  xy (3  z ) (1.1) � (3  z )3  z � 3  x y  xy x y z Khi đó: z  z  � 3  x y  xy x y z (1.2) x3 y 2  x y  xy �3 3 3 3 x y z Để ý rằng: x y z Đồng thời: 3z  z   3( z  1)3  �0 z z Nên (1.2) đúng, BĐT ban đầu chứng minh Vậy k =3 số nguyên dương nhỏ để BĐT ban đầu Dấu xảy x = y = z = Bài Tìm tất số thực k cho bất đẳng thức sau với số thực không âm a, b, c (a  b  c) ab  bc  ca �  k max  (a  b) , (b  c ) , (c  a)  �a  b  c (THPT chuyên Đại học Vinh) Lời giải max  ( a  b) , (b  c ) , (c  a)   ( a  c) Khơng tính tổng qt giả sử a �b �c Khi (a  b  c) ab  bc  ca �  k (a  c) �a  b  c Như vậy, ta tìm k cho : 1 �k � Ta chứng minh Cho c = 0, a = 2b ta (a  b  c) 1 ab  bc  ca �  k ( a  c) �a  b  c �k � với (a  b  c)2 1 ab  bc  ca �  k (a  c)  (k  )(a  c)  (a  c  2b) �0 12 Ta có nên BĐT Đồng thời (a  b  c) 1  k ( a  c) �a  b  c  (  k )( a  c)  (a  c  2b) �0 nên BĐT thứ hai Bài Cho x, y, z ba số thực dương thỏa mãn xyz = Chứng minh bất đẳng thức 1   � 2 (2 x  y  z ) (2 x  y  z ) (2 x  y  z ) 16 2 2 Cho x , y, z không âm thỏa x  y  z  Chứng minh bất đẳng thức � ( x y  y z  z x) �  � x2  � y2 1  � �� z2 1 � � (Bà Rịa – Vũng Tàu) Lời giải Trước hết xin phát biểu không chứng minh bổ bề quen thuộc Bổ đề Co x, y, z > Khi 9( x  y )( y  z )( z  x) �8( x  y  z )( xy  yz  zx) Trở lại toán Theo bất đẳng thức AM-GM , ta có �(2 x  y  z ) 1 � �� (( x  y )  ( x  z )) 4( x  y )( x  z ) Do BĐT ban đầu ta chứng minh �  ( x  y  z ) �( x  y )( y  z )( z  x ) 4( x  y )( x  z ) 16 � � ( x  y  z )( xy  yz  zx ) �( x  y )( y  z )( z  x )( xy  yz  zx ), 3 2 Nhưng điều xy  yz  zx � x y z  theo bổ đề bên Từ ta có điều phải chứng minh Dấu xảy a = b = c = Chúng xin nêu hai cách chứng minh cho câu  Cách 1: Ta có ( x  y  z )( x  y  z )  x  y  z  x ( y  z )  y ( z  x)  z ( x  y ) 2 Áp dụng bất đẳng thức AM- GM x  y  z  ta có x2 y  y2 x  z x � ( x  y  z) Áp dụng bất đẳng thức Cauchy: � 1 � �1 1 � � ��3 �     � � x2  y2 1 z2 1 � � � �x  y  z  � Do ta chứng minh �1 � 1 �� 3�   ��� � �x  y  z  � �2( x  y  z � ۳ 27 4( x  y  z ) 1   x 1 y 1 z 1 27 x2 y2 z2    �3 4( x  y  z ) x  y  z  Áp dụng bất đẳng thức Cauchy-Schwartz ta có � ( x  y  z )2 27 VT �  2 3 x  y  z 4( x  y  z )2 ( x  y  z)2 27  VT �  4( x  y  z ) 2 2 Áp dụng BĐT AM- GM x  y  z � 3( x  y  z )  ta có  27 ( x  y  z )2 ( x  y  z) 18 18     �  3 2 4( x  y  z ) 4( x  y  z ) 4( x  y  z ) 4.3 Từ ta thấy đpcm Cách 2: Ta có ( x  y  z )( x  y  z ) x  y  z x2 y  y z  z x �  3 Do cần chứng minh x yz x yz x yz   �  x2 1 y2 1 z2 Ta có : xy  zx  zx �x  y  z  Do x x 1� x x � � � �  � ( x  y )( x  z ) �x  y x  z � 1 x Do x y z   � 2 2 1 x 1 y 1 z Hay chứng minh yz zx x y   �3 x2  y  z 2 y  z  x2 z  x2  y Ta có : � x y � � 2x2  y  z � Suy : � y2 z2 ��  � x  y x2  z � � yz � � 2x  y2  z2 � Mặt khác: � � zx � � � � 2z  x2  y2 � � 2 � � x y � � � � 2z  x2  y � � � ��3 � � � � yz zx x y � �   2 2 2 � � 2x2  y  z 2 y  z  x z  x  y � � � � yz zx x y � �3 �   2 2 2 � � 2x  y  z 2y  z  x 2z  x2  y � � Suy yz zx   �� ���9 �� �� x y �3 x2  y  z 2 y  z  x2 z  x2  y Do ta có đpcm Bài Cho a, b, c > thỏa mãn điều kiện a + b + c = Tìm giá trị lớn biểu thức T ab bc ca    3a  4b  5c 3b  4c  5a 3c  4a  5b ab(a  2c)(b  2c) (Bắc Ninh) Lời giải Ta có ab 2ab � 5ab ab � � �3z  4b  5c  �5(a  b  2c)  (a  3b) �36 � � � a  3b � � a  b  2c Bây ta chứng minh ab ab 1� �a  b  2c �4 Ta có : ab ab �  (a  b  c)  �a  b  2c �4 � �  �b  c � � ca � Nên điều ta chứng minh: ab �a  3b �4 Để ý ab � ab 3ab � 1 �  �a  �3a   (a  b  c)  4 � 16 �a  3b �16 � � �b � a  ۣ ab �3a  4b  5c 9 (5  )  18 4 (1.3) Mặt khác 2  � ab(a  2c)(b  2c ) (ab  2bc)(ab  2ca ) 2(ab  bc  ca )  3 �   3(ab  bc  ca ) (a  b  c ) 27 (1.4) 77 77 T�   27 108 Vậy GTLN T 108 a = b = c = Từ (1.3) (1.4) ta : Bài Cho a, b, c số thực dương Tìm giá trị nhỏ biểu thức: P 1344 2016  a  ab  abc abc (Bến Tre) Lời giải Ta có : a  ab  abc  a  a a a a b  b.4c �a   b  (  b  4c )  (a  b  c)  4 4 1008 2016 � � P�   1008 �  1� 1008 �1008 abc abc � abc � Vậy GTNN P -1008 a 16 ,b  ,c  21 21 21 Bài Cho số thực dương x, y, z Chứng minh  (�x) x2 x yz �  �y  z �( y  z )  x yz yz �۳ yz xy x y zx zx ( x  y )2 � xy (đúng) Bài Cho số thực dương a, b, c thỏa mãn abc = Chứng minh : a b c 3   � bc ac ab a  b3  c  (Đồng Nai) Lời giải Ta có : a 9 � b  c  � bc(b  c) � bc(b  c) � 3( bc(b  c)) � � � 3 a  b3  c3  3abc  3 a  b3  c  Bài Cho a, b, c �0 Tìm giá trị nhỏ P a b c   bc ac a b (Hà Nam) Lời giải a a  � a  a Điều tương đương Chuẩn hóa a + b + c = Ta chứng minh b  c với a (2a  3) �0 , hiển nhiên Cộng lại ta kho có số hai số P �  Vậy GTNN P Nhận xét Một số bạn giải sau: Ta có a 2a 2a � �� 2 bc a bc a (b  c ) phải xét trường hợp có số 0., để ý nhân tử mẫu phân thức cho số, số phải khác P� Bài 10 Cho a, b , c > thỏa mãn ab + bc + ca + 2abc = Tìm giá trị nhỏ P 1    2(a  b  c) a b c (Hà Nội) Lời giải abc , ta cố gắng chứng minh BĐT Dự đoán GTNN P đạt P 1    2(a  b  c ) �3 a b c Từ giả thiết suy tồn số x, y, z > cho a x y z ,b  ,c  yz zx x y BĐT cần chứng minh trở thành �x yz zx x y y z �   �2 �   � x y z �y  z z  x x  y � Để ý �x yz zx x y y z �   �4 �   � x y z �y  z z  x x  y � Nên BĐT ta chứng minh x y z   � y z z x x y Nhưng dây BĐT Nesbitt quen thuộc, BĐT ban đầu x y z ,b  ,c  yz zx x  y kinh điển việc đổi biến Nhận xét Cách đặt hóa để chứng minh BĐT, giúp đưa dạng tốn quen thuộc Ngồi cách khác cho loại giả thiết tương tự Cụ thể sau, x , y, z số dương : a   x  y  z  xyz   2cosA, y  2cosB, z  2cosC với A, B, C ba góc tam giác bc ca a b xyz  x  y  z   x  ,y ,z  a b c với a, b, c > Bạn đọc dễ dàng kiểm tra cách đặt Ngồi số tốn khác liên quan đến cách đổi biến lượng giác : 2 (USA 2001) Cho a, b, c không âm thỏa mãn a  b  c  abc  Chứng minh rằng: �ab  bc  ca  abc �2 2 2 (Iran 2002) Cho a, b, c > thỏa mãn a  b  c  abc  Chứng minh : a  b  c �3 Bài 11 5 Cho số thực dương a, b, c dương thỏa mãn a  b  c  Chứng minh a 6b  b 6c  c a �3 (Hà Tĩnh) Lời giải 5 Đặt x  a , y  b , z  c x + y + z = BĐT cần chứng minh tương đương với xy xy  xy yz  zx zx �3 Thep BĐT AM – GM �xy x, y ,z  xy ( x  y  3) xy ( x  y  x  y  z ) xy ( x  y )  xyz xy �� � � 5 cyc cyc x, y,z 2( x  y  z )( xy  yz  zx)  3xyz 6( xy  yz  zx )  3xyz  5 Mặt khác theo BĐT Schur ( x  y  z )3  xyz �4( x  y  z )( xy  yz  zx)  32  xyz �4.3( xy  yz  zx) Suy 6( xy  yz  zx) _ 3xyz 6( xy  yz  zx)  4( xy  yz  zx)  � 5 ( x  y  z)2  2( xy  yz  zx)   � 3 5 Vậy BĐT ban đầu chứng minh Dấu xảy a = b = c = Trong T, số tự nhiên thuộc khoảng [1; 2016] xuất hai lần, có 2016 dấu cộng 2016 2016  1008 dấu trừ Như vậy, T đạt giá trị lớn 2016 dấu cộng thuộc số lớn nhất, �2016 1008 � Tmax  ��i  �i �  1008.1008   2.10082 i 1009 i 1 � � 2016 dấu trừ thuộc 1008 số lại Vậy ,đạt (f(1),f(2), ,f(1008)) hoán vị (1009,1010, ,2016) ( f  1009  , f  1010  , , f  2016  ) hoán vị (1,2, , 1008) f  i  i Thật vậy, giả sử tồn i < 1009 cho f(i) < 1009 Khi từ xuất số bé 1009 mang dấu cộng, vơ lí Từ điều kiện dấu bằng, ta tính số hốn vị là: 1008! 1008! = (1008!)2 Trước hết xin phát biểu chứng minh bổ đề sau  n �2  , có 22 + đoạn thẳng nối hệ điểm có Bổ đề 16 Trên mặt phẳng xét 2n điểm tam giác Đây số cạnh cần có để ln tồn tam giác Chứng minha 4.3 6  với n = 2, xét tập điếm {A, B, C, D} Ta có số cặp hai điếm Suy có cặp khơng nối AB  Giả sử bổ đề với n = k, xét n = k + Giả sử tồn cách nối cho không tồn tam giác xét tập S có 2k điểm tập hợp 2(k + 1) điểm xét cho hai điểm lại, đặt A B, nối với Nếu có k2 đoạn thẳng có hai đầu mút thuộc S S có tam giác theo giả thiết quy nạp xét ba (A,B,X) với X chạy S Mỗi ba có nhiều cạnh, khơng tính AB có nhiều cạnh Khi cho X chạy S ta nhiều 2k đoạn thẳng Vậy k  2k   (k  1) � k  1  số đoạn thẳng nhiều , vơ lí Nếu nối (k+1 )2 đoạn thẳng, ta có cách nối sau: Chia S thành hai nhóm (k + 1) điểm, điểm nối với tất điểm khác nhóm với với cách nối này, khỉ chọn điểm bất kì, theo nguyên lí Dirichlet, ln có hai điểm chung nhóm, khơng nối với  n �2  Vậy bổ đề với n = k+1 Theo nguyên lí quy nạp, bổ đề với A Quay lại toán Gọi tập hợp đoạn thẳng cần nối A với nhỏ Điều kiện đề tưong đưong với không tồn ba điểm đôi không nối với Gọi S tập hợp tất �42 � S / A �� � �2 � Suy cặp điểm Khi theo bổ đề ta có 42.41 �42 � A  S S/A �  � � 420 �2 � Bài Xung quanh bờ hồ hình tròn có 2017 liễu Người ta dự định chặt bớt liễu cho khơng có liễu kề bị chặt Hỏi có cách thực khác ? 12k  k ��* Một họp có người, người bắt tay vối 3k + người khác Biết với cách chọn cặp hai người (A;B) số người bắt tay với hai người A m  m  N *, m 3k   B ln Hỏi họp có người ? (Bắc Ninh) Lời giải Đánh số liễu từ đến 2017 theo chiều kim đồng hồ Trước hết ta tìm số nghiệm nguyên dương phương trình a+b+c+d+e = 2012 Dễ thấy số cách chặt thỏa yêu cầu với chắn bị chặt (ví dụ, số 1) số nghiệm nguyên dương phương ttình 2011! 2011!  4! 2011   ! 4!2007! 2011!2017 Do có 2017 cách chặt bị đếm lần ta nên số lượng cách chặt là: 4!2007!5 cách Ta đếm hai cách số bắt tay họp Cách đếm thứ nhất: Do người bắt tay k + lần bắt tay thực 12k  3k    6k  3k   bới hai người nên số băt tay là: 12k  12k  1  6k  12k  1  Cách đếm thứ hai: số lượng nhóm hai người Với nhóm hai người theo đề có m người bắt tay vối hai người từ nhóm hai người ta đếm có 2m bắt tay (khơng tính bắt tay hai người có) xét hai người bắt tay A B Dễ thấy bắt tay A B đếm 2(3 k + 5) nhóm hai người bao gồm (A, XB) (B, XA) với XA chạy tập người bắt tay với A khơng tính B XB chạy tập người bắt tay với B khơng tính A Vậy tổng số bắt tay 6k  12k  1 2m 6km  12k  1   3k   3k  6km  12k  1  6k  3k   3k  Từ hai cách đếm trên, ta phương trình nghiệm nguyên dương: Giải phương trình ta k = 3, m = Vậy họp có 12k = 36 người Bài  Trên mặt phẳng cho 2017 điểm cho với ba điểm ta ln tìm hai điểm để đoạn thẳng tạo thành có độ dài bé Chứng minh ln tồn đường tròn có bán kính chứa khơng 1009 điểm cho (Bình Dương) Lời giải Gọi tập hợp điểm cho S Xét điểm A S TH1: Mọi điểm lại tập có khoảng cách với A bé Đường tròn tâm A bán kính đường tròn cần tìm Kết tốn trường hợp TH2: Tồn điểm B tập hợp điểm lại cho AB �1 Xét ba điểm (A,B, X) với X chạy S/{A,B} DO AB �1 nên AX < BX < Theo nguyên lí Dirichlet, hai đường tròn (A, 1) (B, 1) chứa 1008 điểm thuộc S/{A,B} Đường tròn đường tròn chứa 1009 điểm thuộc S tính thêm A,B Kết toán Bài 10 2016 , , , 2016 , M ỗ i lần thưc hiên trò chơi cho phép ta xóa Trên bảng cho số 2016 2016 hai số a, b thay vào số a + b- 3ab Hỏi sau 2015 lần thực phép xóa, số lại bảng số nào? (Bình Dương) Lời giải 672 672 672  Ta xét số 2016 Nếu ta xóa số x số 2016 , ta nhận lại 2016 672 672 672 672  x3 x 2016 2016 2016 Vậy số 2016 diện bảng , đáp án tốn Bài 11 Trên vng bảng 9x9, ta đặt châu chấu Giả sử sau tiếng gõ, châu chấu nhảy sang ô bên cạnh hàng cột Chứng minh sau tiếng gõ có hai ô Trên mặt phẳng tọa độ Descartes Oxy, có châu chấu tọa độ (x,y) x, y �� Với N số nguyên dương cho trước, châu chấu nhảy từ điểm nguyên A đến điểm nguyên B độ dài AB N Hỏi châu chấu nhảy đến điểm nguyên sau hữu hạn bước nhảy không N = 2025 sao? (Điểm ngun điểm có tung độ hồnh độ số ngun) (Bình Dương) Lời giải Ta tô màu bảng cho đen trắng xen kẽ với góc màu đen Dễ dàng tính số đen nhiều số ô trắng ô sau tiếng gõ, châu chấu ô đen nhảy sang ô trắng ngược lại, từ ô trắng nhảy sang ô đen Từ suy sau tiếng gõ, số châu chấu ô trắng nhiều số ô trắng đơn vị Khi theo ngun lí Dirichlet, có châu chấu ô Theo đề bài, châu chấu vị trí (a, b) đó, nhảy tới vị trí (a +m,b + n) với m, n số nguyên thỏa m2 + n2 = 2025 Do 2025M3 nên m, nM3 Vậy số dư chia hồnh độ (tương tự, tung độ) cho khơng đổi Vậy kết luận tốn khơng Bài 12 Trong mặt phẳng cho n �2 đường thẳng đôi cắt khơng có ba đường đồng quy Các đường chia mặt phẳng thành miền hữu hạn vơ hạn Chứng minh ta đánh dấu miền số nguyên thỏa mãn ba điều kiện sau: i Các số khác ii Trị tuyệt đối số không lớn n iii Mỗi đường thẳng cho phân mặt phẳng làm hai phần mà tổng số miền thuộc phần (Đà Nẵng) Lời giải Nhận xét miền có tối đa n cạnh nên số đỉnh miền khơng vượt q n Ta có cách đánh số sau: Chọn miền bất kì, đánh số cho miền số đỉnh Sau ta đánh số miền lại theo quy tắc: số đánh dấu có giá trị tuyệt đối số đỉnh miền số đánh dấu hai miền chung cạnh trái dấu Ta chứng minh ln đánh số cách  Với n = hiển nhiên mệnh đề  Giả sử với n = k mệnh đề đúng, xét k + đường thẳng thỏa mãn yêu cầu đề Ta bỏ qua đường thẳng đánh số theo cách Theo giả thiết quy nạp, ta đánh số Bây ta xét thêm đường thẳng br qua Ta chọn bên bất kì, giữ nguyên dấu miền bên ỏ miền bị phân chia, bên phía chọn ta giữ dấu miền ban đầu, phía lại trái dấu Các miền khơng bị phân chia phía khơng chọn đổi dấu Dễ thấy cách chọn dấu âm, dương thỏa mãn cách đánh số nêu Vậy mệnh đề với n = k + theo nguyên lí quy nạp, mệnh đề với n Cách đánh số thỏa mãn điều kiện đề Thật vậy: Điền kiện (i) hiển nhiên thỏa mãn Điều kiện ( i i ) thỏa mãn dựa nhận xét ban đầu Đối vối điều kiện ( i i i ) : Dễ thấy đỉnh thuộc miền không nằm đường thẳng xét (loại 1), thuộc miền nằm đường thẳng xét (loại 2) Tổng miền "dương" thực chất đếm số đỉnh loại nhân cộng cho số đỉnh loại Tương tự với tổng miền "âm" Vậy tổng miền thuộc phía đường thẳng xét Điều chứng tổ điều kiện (iii) thỏa mãn Vậy toán chứng minh Bài 13 Cho bảng ô vuông 2017 X 2017, người ta điền vào ô bảng số nguyên từ đến 2017 cho số điền vào bảng lần Chứng minh tồn hai số cạnh bảng (tức thuộc hai chung cạnh) có hiệu khơng nhỏ 2017 Tìm k ��* nhỏ cho tồn cách điền để hiệu hai số cạnh bảng khơng lớn k (Đà Nẵng) Lời giải Ta chứng minh phản chứng, giả sử tồn cách đánh số cho hai số cạnh bảng có hiệu nhỏ 2017 Xét số k cho �k � 2017  2017 Gọi Ak = {k+1, k + 2, , k + 2016}, Bk = A  2016 {1,2, ,k}, Ck = {k + 2017, k + 2018, ,20172} k nên tồn nhắt cột (tương tự, cạnh) không chứa phần tử thuộc Ak Gọi tập hợp số thuộc vào cột hàng Dk Cũng Ak  2016 nên Dk khơng thể có hai phần tử kề không thuộc D  4033 tập hợp, suy Dk tập Bk Ck Dễ thấy k B 1  Với k = 1, k nên D1 �C1 C 1 D �B20172  2017 Với k  2017  2017 , k nên 20172 2017 D �Cn D �Bn 1 h �Cn Vậy tồn n cho n n 1 Vậy tồn số h cho h �Bn 1 Khi ta có n  2017 �h �n  , vô lý! Theo câu 1, ta có k �2017 Ta có cách đánh số sau: Hàng thứ i đánh số từ trái sang phải 2017(i - 1) + j với j vị trí hàng Dễ thấy cách đánh số thỏa mãn hiệu hai số cạnh khơng vượt q 2017 Vậy số cần tìm 2017, với cách đánh số Bài 14 a , a , , an Tìm số nguyên dương n nhỏ cho: Với n số nguyên dương đôi khác a  a j �2017  , a j  i, j � 1, 2,3, , n nhau, tồn hai số để i với (a, b) ước chung lớn hai số nguyên dương a, b (Đồng Nai)  Lời giải Dễ thấy 2017 số nguyên tố xét 1008 số nguyên dương Ta có hai số ngun dương dãy có tổng không 1008 + 1007 = 2015 < 2017 Vậy n > 1009  a , a , , a1 009  Ta chứng minh 1009 số cần tìm Giả sử tồn cách chọn 1009 số cho không thỏa điều kiện đề a  Giả sử tồn số i, j cho chia hết cho 2017 j khơng chia hết cho 2017 a �a � � a , a j �� i  a j �2017 � i , a j �  , a j   � � �2017 � 2017 suy �2017 �mâu thuẫn Khi ta có với giả thiết phản chứng Vậy dãy tất chia hết cho 2017, tất không chia hết cho 2017 Dễ thấy ta tìm số khơng thỏa điều kiện, chia số cho ước chung lớn chúng, ta thêm số khơng thỏa điều kiện, ta cần xét trường hợp tất không chia hết cho 2017 a  a j M2017  Giả sử tồn hai số i , j cho i Khơng tính tổng qt, giả sử  a j  a j 2017b  � �2017 , a j   b, a j   b, a j  a  a j � , a j   a j  2017b, b ��  Ta có suy i Vậy 1009 số đôi không dùng số dư chia cho 2017 Ta có nhóm (1,2016); (2,2015); ; (1008,1009) Theo nguyên lí Dirichlet, phải tồn hai số i,j cho số a a  a j M2017 a dư chia i j cho 2017 1008 nhóm Khi ta có i  a j  2017c 2017c � �2017  2017c, a j   c, a j  a  a j  2017c  a ,a  Đặt i Vậy ta có i j Suy  a j �2017  , a j  , mâu thuẫn vối giả thiết phản chứng! Vậy giả thiết phản chứng sai, ta có điều cần chứng minh Bài 15  a , a , , a10  số 1,2,3,…,10 cho  a2i với �i �5 Có hốn vị a j  a2 j 1 với �i �4 (Đồng Nai) Lời giải Từ điều kiệnn để ta a1  a2 , a3 ; a2  a4 , a5 ; a4  a8 , a9 ; a5  a10 ; a3  a6 , a7 Dễ thấy a số lớn  a2 , a4 , a5 , a8 , a9 , a10   a3 , a6 , a7  nhất, a1 = 10 Chia số lại thành hai nhóm Dễ thấy hai nhóm khơng ảnh hưởng đến xét đến điều kiện đề Ta có số cách chọn 9! phần tử nhóm thứ 3!6! Khi chọn xong phần tử cho nhóm 1, lại phần tử cho nhóm Do nhóm 2, phần tử a3 phần tử lớn nhất, a6 a7 không ảnh hưởng đến nên số cách cho nhóm Trong nhóm 1, để ý a2 phần tử lớn nhóm Các phần tử lại chia thành hai  a ,a ,a   a ,a  nhóm 10 Rõ ràng hai nhóm không ảnh hưởng đến Vậy số cách chọn phần tử cho hai nhóm 5! là: 2!3! Nhóm nhỏ thứ tưong tự nhóm hai nên số cách a  a10 Nhóm nhỏ thứ hai có nên có số cách 5!9!  3360 Vậy số hoán vi thỏa mãn là: 2!3!3!6! 2!3!3!6! Bài 16 x , x , x � 0;1; 2; ;7 x   x1 , x2 , x3  �A  x ,x ,x  Gọi A tập với Bộ gọi trội y   y1 , y2 , y3  �A x �yi , i  1; 2;3 n ��* x �y i Khi ta viết x > y Tìm cho tập có n ptử A chứa x > y (Hà Nam) Lời giải Xét tập B A cho khơng có hai phần tử B mà phần tử trội phần tử lại, |B| đạt giá trị lớn Xét bảng 8x8 đánh số hàng từ xuống từ đến cột từ trái sang phải từ đến Nếu B có phần tử x = a, b, c ta đánh dấu (a,b) Dễ thấy khơng có đánh dấu q lần Thật vậy, giả sử có (a, b) đánh x   a, b, x3   y   a, b, y3  dấu lần, tập B có tương ứng hai phần tử , vơ lí! Vậy số đánh dấu |B|  a ,b   a ,b  Ta gọi ô 1 bé (tương tự, lớn hơn) ô 2 a1 �a2 b1 �b2 (tương tự, a1 �a2 b1 �b2 ) Dễ thấy, bảng khơng thể có  a1 , b1  ; ;  a9 , b9  cho  , a j    1, b j 1  , i  1,8 , tồn tại, theo ngun lí Dirichlet phải tồn hai số i < j c  cj x  xj cho i dẫn đến j Gọi S1 tập tất ô đánh dấu mà ô đánh dấu lớn hơn, , Si tập tất ô đánh dấu mà vối phần tử thuộc Sị+1, có phần tử thuộc Si+1 lớn S 0 khơng có phần tử không thuộc Sk với k �i lớn Ta có | với i bất kì, xét phần tử thuộc Si , gọi mi giá trị lớn a + b với (a,b) thuộc Si, gọi phần tử m  mi 1 thuộc Si (ai , bi) Để ý (a, b) < (c, d) a + b < c + d Vậy nên ta có i  c ,d   a ,b  c �ai d �bi c  Xét phần tử i i thuộc Si khác i i Rõ ràng i i Ngoài ra, i S �m d  bi c  d  bi m �8 i ngược lại, i i Vậy i i Si �16  mi m 8 i | B | �Si �5         48 i 1 Dễ dàng thấy đ ó Dấu xảy ra, chẳng � a  b � 12 hạn ô đánh dấu (a, b) thỏa ngược lại, ô thỏa điều kiện đánh dấu Vậy ta có n �49 , hay nmin  49 Bài 17 S   1, 2,3, , 2016  a , a , , a2016  tập S cho Cho tập Hỏi có hốn vị 2  a1  a2  a3   ak  Mk với k  0,1, 2, , 2016 (Hà Tĩnh) Ta chứng minh quy nạp Lời giải ak  a1 , a2 , , a k ak  maxa1a2 , , ak 2016   �ai  2016.2017 với k �4 2015 �ai M2015 a , a , , a 2016 Với k = 2016, hiển nhiên nên i 1 Ta lại c ó i 1 a � 1; 2016 nên a2016  1M2015 Vậy 2016 Mệnh đề với k = 2016  a , a , , an1  Giả sử mệnh đề từ k = 2016 đến k  n �5 Xét k = n+1 Hiển nhiên n2 2�i   n  1  n   M n  1 hoán vị n - số liên tiếp Ta có i  nên ta  a , a , , a n 1  hoán vị  1, 2, , n  1 Tương tự ta có xem n 1 2�ai  n  n  1 i 1 n2 2�ai Mn  i 1 an 1  1M n   an 1  an 1  n  n 1 2�ai  n  n  1 n2 2�ai Mn  a  1M n   Do i 1 nên n 1 an 1  an 1  n  Vậy mệnh đề với k = n - Theo ngun lí quy nạp, mệnh đề vói k > Gọi Sk số hoán vị (l,2, ,k) thỏa mãn yêu cầu đề Theo mệnh đề chứng minh, ta có S k 1 S k với k �3 Dễ dàng tính S3 = Vậy số lượng hoán vị thỏa yêu cầu 22014 Bài 18 Xét tập hợp S gồm 2016 số nguyên dương Gọi A, B,C tập S, đôi A  B  C  672 không giao cho A �B �C  S Chứng minh tồn số a, b, c thuộc tập A, B, C mà số tổng hai số (Tp.HCM) Lời giải Giả sử phản chứng, tồn cách chọn tập A, B, C thỏa mãn điều kiện đề cho không tồn số a, b, c thuộc A, B, C mà số tổng hai số lại Khơng tính tổng quát, giả sử �A Theo giả thiết phản chứng, tồn hai số liên tiếp mà số thuộc B, số lại thuộc C Ta viết lại số thành chuỗi chữ a, b, c theo qui tắc số i thuộc tập A chữ vị trí thứ i chữ a, tương tự với B,C Không tính tổng quát, giả sử chữ cuối chữ c Ta chứng minh tồn chuỗi acc Giả sử khơng tồn chuỗi Khi số lượng chữ a cần có để ngăn cách chữ c 672 Tuy nhiên ta cần thêm chữ a để ngăn cách 672  A �672  chữ c với 672 chữ b, vơ lí Vậy chuỗi acc có tồn Giả sử chữ a chuỗi vị trí x Ta chứng minh tồn chuỗi bba Giả sử khơng tồn chuỗi Khi chữ cuối chữ c chữ b phải ngăn cách với chữ c chữ a nên tồn chữ a vị trí lớn vị trí tất chữ b cần 673 chữ a Suy điều vơ lí 673 �672 Vậy tồn chuỗi bba Giả sử chữ b chuỗi vị trí y TH1 : x > y Xét hiệu ( x - y ) D o x �A v y �B nên theo giả thiết phản chứng, (x - y) không i 1 thể thuộc c Tương tự với x + l v y + , x + v y + t a có (x - y) khơng thể thuộc A B Suy (X - y) không thuộc tập cả, vơ lí TH2: x < y Xét hiệu (y - x) tương tự trên, ta suy (y - x) khơng thuộc tập cả, vơ lí Vậy giả thiết phản chứng sai, ta có điều cần chứng minh Bài 19 A   a1 , a2 , , a15  Cho tập hợp gồm 15 phần tử Chúng ta tạo tập hợp mà tập hợp chứa hay nhiều phần tử A (có thể sử dụng tất phần tử tập A) số phần tử tập hợp tạo thành phải bội số nhỏ có tập hợp Có tập hợp hợp tạo thành? Chẳng hạn  a2 , a4 , a8  ,  a6  , tập hợp thỏa mãn yêu cầu tốn (Khánh Hòa) Lời giải 15 � � 1 � � n � � Xét số tự nhiên n cho < n < 15 Dễ thấy [1; 15] có N số khác n bội số n số tập hợp thỏa diều kiện đề vối n số nhỏ sơ tập tập hợp bội số n nhỏ 15, bằng: 15 �S i Sn  15 � � 1 � �n � �  214  26  24  2.22  2.21  8.20  16492 Số tập hợp thỏa điều kiện đề là: i 1 Bài 20 M  {1; 2}(n �N *) Cho tập n Gọi X tập M15 cho tích ptử X khơng phải số phương max X Tìm Gọi Y tập gồm có 15 phần tử tập M25 Tập I' gọi tập "tốt" khơng tồn phần tử mà tích chúng số phương Tính số tất tập "tốt" (Lạng Sơn) Lời giải Ta chia số từ đến 15 cho ước phương lớn Khi ta số 1,2,3,1,5,6,7,2,1,10,11,3,13,14,15 Giả sử |X| > 10  Nếu X chứa số 1: Khi X khơng có hai phần tử có tích số phương nên X chứa tối đa số 1, chứa tối đa số 2, chứa tối đa số  Nếu X chứa số X khơng thể chứa hai số 10, chứa hai số X �15   10 X 14 Khi , mâu thuẫn với giả thiết quy nạp Vậy không chứa 2, mà X  10 nên V = {1,1,3,5,6,7,10,11,13,14,15}, ba số (3,5,15) có tích số phương Vậy X khơng chứa số 1, X chứa ba số X �15   10 (2,7,14) (3,5,15) nên X �10 Vậy Đưa số lại ban đầu, ta cách chọn phần tử cho X : X = {1,3,4,5,6,7,10,11,13,14} Tương tự trên, ta thay số từ đến 25 thương chia cho ước phương lớn nó, ta 16 số 1,2,3,5,6,7,10,11,13,14,15,17, 19,21,22,23 với số lần xuất số 5,3,2,2,2,1,1,1,1,1,1,1,1,1,1,1 Theo điều kiện đề bài, số ta lấy phải khác nên số cách chọn 15 phần tử (cũng số đề yêu cầu tính) là: �1 � 5.3.2.2.2 �    11� 1564 �5 � Bài 21 Một số nguyên dương k gọi " đẹp" phân hoạch tập hợp số nguyên dương i � 1; 2; ; k  Z* thành tập hợp A1,A2, ,AK cho với số nguyên dương n �15 với tồn hai sô thuộc tập A i có tổng n Chứng minh k = đẹp Chứng minh k �4 không đẹp (Nghệ An) Lời giải Ta có cách phân hoạch sau: Ai   1; 2;3;12; ;3k ;  , A2  A3   4;5;6;11; ;3k  2 ,  7; 8; 9; 10; ;3k  1;  Với n = 15, n = 16 n = 17 Đối với 3+12=15; 1+15=16; 2+15=17 Đối với A2: 4+11=15; 5+11=16; 6+11=17 Đối với A : 7+8=15; 7+9=16; 8+9=17 k �6 :  3k  3   3k  Với n > 18: Đối với Ai Khi n = 3k với Khi n = 3k + với k �6 :  3k    3k  k �6 :  3k    3k  Khi n = 3k + v i Tương tự với A2 A3 Vậy k = số đẹp Giả sử phản chứng, tồn cách phân hoạch thỏa đề với k �4 xét tập S = {1; ; … ; } , 15 < 44 nên phải tồn số i cho Ai chứa không số thuộc S Không  k ;15  k tập tính tổng quát, giả sử số Theo điều kiện đề bài, tồn k �7 cho A1 Đặt số lại h Ta có k + h = 16 15 - k + h = 16 Suy k phần tử bé A1 Giả sử k > 1: Trong A1 phải tồn số có tổng 17 Do phần tử nhỏ Ai lớn nên ta có 15 - k + h = 17 k + h = 16 Giải hệ phương trình ta k = 7,h = Nhưng khơng tồn số thuộc Ai có tổng 18 18 - < 18 - < 18 - < 15 Vậy ta có k =  1; 14; 15; 16; 17; 18; ; 27 �A1 Giả sử 15 �A1 Ta xét số từ 17 đến 28, ta Khi ta xét số 27, 27: = 13.5 nên với i > 1, không tồn hai phần tử thuộc Ai có tổng 27, vi phạm điều kiện �A1 Vậy 15 khơng thuộc A1 Khi Bài 22 Tìm kmax ��* cho ta phân hoạch tập hợp số nguyên dương thành k tập hợp  - A ,A , ,A k thỏa mãn với n ��*, n  14 , trogmỗi tập băng n Aii  1, k tồn sơ có tống (Ninh Bình) Lời giải Bài hoàn toàn tương tự 19 đề Nghệ An Bạn đọc dùng cách giải 19 để giải toán Bài 23 Một hàng bưởi Đoan Hùng gồm 17 thẳng hàng đánh số theo thứ tự số tự nhiên từ đến 17 Ban đầu có đậu để hút mật hoa Sau đó, có hai ong bay sang hai bên cạnh để tìm hút mật theo hai chiều ngược Hỏi sau có hay khơng trường hợp mà Khơng có ong có số thứ tự chẵn Có ong cuối (Phú Thọ) Lời giải Có thể xảy trường hợp này, chẳng hạn, sau thứ nhất, ong chuyển sang 3, ong chuyển sang 3; ; sau thứ 4, ong 14 chuyển sang 15, ong 16 chuyển sang 15 Lúc này, khơng có ong có vị trí chẵn Đánh số ong vị trí bưởi mà đậu Gọi S tổng tất 17 S  �i  17.9 i 1 ong Ban đầu ta có Khi ong bay sang bên cạnh, bay hướng số 1, số gán cho giảm 1; ngược lại, bay hướng số 17, số gán cho tăng Do sau có hai ong bay sang bên cạnh ngược hướng nên s không thay đổi Vậy có ong cuối cùng, S > 17.9 = S, vơ lí! Vậy khơng thể xảy trường họp Bài 24 Cho số nguyên dưong n �4 lìm số lớn cặp gồm hai phần tử phân biệt tập X n   1, 2, , n cho tổng cặp khác số nguyên khác không vượt n (Quảng Bình) Lời giải Giả sử k số cần tìm Do có 2k số phân biệt k cặp tìm nên tổng chúng khơng nhỏ hơn: k(2k+ 1) Do tổng cặp khác không vượt n nên tổng 2k số khơng lớn hơn: k  k  1 k nk  k  k  1 k  2k  1 �k  n  k   Vậy 2n  k� Suy Bài 25 Giả sử S tập họp hữu hạn điểm mà điểm tơ màu đổ A , A , , A68 xanh Gọi tập tập s mà tập chứa điểm thỏa mãn đồng thời hai điều kiện sau: i Mỗi tập A1 , A2 , , A68 chứa điểm màu đổ ii Với ba điểm S, tồn xác tập Ai chứa điểm Hỏi: 1/ Tìm số phần tử tập S 2/ Tồn hay không tập Ai chứa điểm đỏ Vì ? (Quảng Ninh) Lời giải Số tập có ba phần tử tập hợp năm phần tử là: 5!  10 3!   ! Do tập ba phần tử S tập xác tập Ai nên số lượng tập S  S  1  S    680 ba phần tử S băng: 68.10 = 680 Suy Giải phương trình với S S  17 số tự nhiên, ta Vậy số phần tử S 17 Giả sử phản chứng, không tồn tập Ai chứa điểm đỏ Gọi tập tập Ai F Ta có nhận xét sau: Ai ǣA j A �Aj   với i �j Điều tồn tai i �j cho i có ba điểm tồn Ai Aj, vi phạm điều kiện  Bộ hai điểm (A, B) xuất phần tử F Xét ba (A, B, X) với X chạy S/{A, B}, có 15 ba ba phải thuộc vào phần tử F Mà tập hợp Ai chứa (A, B) chứa ba Nên có 15:3 = phần tử F chứa (A, B) Để ý tập hợp có hợp băng S Xét hai (A, B) vối A, B tô màu xanh Mỗi tập Ai chứa hai phải chứa điểm đỏ nên có điểm đỏ thuộc S.(*) Xét ba điểm đỏ đó, giả thiết phản chứng nên ba phải thuộc tập Ai có hai điểm lại màu xanh Ta lại xét hai điểm xanh tương tự trên, suy có điểm đỏ  Có điểm đỏ S Thật vậy, giả sử có điểm đỏ, xét hai điểm đỏ tương tự (*), theo nguyên lí Dirichlet, tồn tập Ai chứa hai điểm đỏ thêm hai điểm đỏ khác điểm đỏ lại, mâu thuẫn với giả thiết phản chứng!  Không tồn i cho Ai chứa hai điểm đỏ Thật vậy, giả sử tồn i vậy, xét hai điểm đỏ thuộc Ai tương tự trên, ta suy tồn tập Aj chứa điểm đỏ, mâu thuẫn! 7!  35  68 Số lượng phần tử F chứa điểm màu đỏ 3!(7  3)! suy tồn điểm đỏ R điểm màu đỏ tập Ar Đặt Ar = R, G1, , G4 Xét hai (R, Gi), A dựa nhận xét 2,3 4, tồn thêm tập ri chứa hai xét nhận R điểm đỏ Cho ị chạy từ đến 4, dựa nhận xét 1, có phần tử F nhận R điểm màu đỏ Để ý (R, G) vối G điểm màu xanh thuộc S "tạo ra" hoặc tập hợp thuộc F nhận R điểm đỏ Vậy R "tạo ra" nhiều 10.2: = phần tử F Vậy R "tạo ra" phần tử F Từ 35M5 điểm đỏ "tạo ra" lượng chia hết cho phần tử F có điểm đỏ nên ta có 68M5 , vơ lí! Vậy giả thiết phản chứng sai Câu trả lời là: tồn tập Aị chứa điểm đỏ Bài 26 Cho tập hợp S = {1;2;3; ;2016} Hỏi có tập S có phần tử mà chúng độ dài cạnh tam giác có chiều dài cạnh lớn 1000 ? Cho hình vng có cạnh Bên hình vng có n(n ��*) hình tròn có tổng diện tích lớn hon n -1 Chứng minh tồn điểm hình vng nằm tất hình tròn (Quảng Trị) Lời giải Số lượng tập ba phần tử s thỏa yêu cầu số lượng tập hai phần tử {a, b} S thỏa �a, b �1000 a + b > 1000 Không tính tổng quát, giả sử a>b Suy 2a  a  b �1000  a  500 Ta có: b < a b �1001  a Suy số giá trị b thỏa điều kiện với 500  a �1000 a   1001  a   2a  1001 là: 1000 500 1000.500 a  1001   250000   � 2i  1  � a  501 i  Vậy kết toán Do hình tròn nằm hồn tồn hình vng nên diện tích lớn hình  �1 � S  � �.  �2 � tròn Gọi O tâm hình vng Giả sử có đường tròn khơng chứa O Do điểm hình vng �2�  S � �4 � �.  � � cách O xa khoảng diện tích đường tròn nhỏ  Giả sử tất hình tròn chứa O Vậy O điểm cần tìm Mệnh đề cần chứng minh  Giả sử có hình tròn khơng chứa O Khi tổng diện tích đường tròn lại lớn    8 n 1    n  1  n    n  Suy 8  2 Vậy ta cần xét trường họp n = n = - Với n = 1: Do hình tròn phải có diện tích lớn nên phải tồn điểm hình vng thuộc hình tròn Mệnh đề cần chứng minh - Với n = 2: Do diện tích hình vng mà tổng diện tích hai hình tròn lớn nên hiển nhiên phải tồn điểm nằm hai hình tròn Vậy mệnh đề cần chứng minh Vậy ta có điều cần chứng minh Bài 27 Cho tập hợp E = {1;2;3;4;5} Gọi M tập hợp tất số tự nhiên có ba chữ số, chữ số đôi khác thuộc E Chọn ngẫu nhiên số thuộc M Tính xác suất để số chọn có tổng chữ số 10 (Thái Nguyên) Lời giải  Số lượng số thuộc M có ba chữ số là: 5.4.3  Số lượng số có bốn chữ số thuộc M là: 5.4.3.2 Số lượng số có năm chữ số thuộc M là: 5.4.3.2.1 Vậy theo ngun lí cộng, ta có |M| = 5.4.3 + 5.4.3.2 + 5.4.3.2.1 = 300 Số lượng số thuộc s có ba chữ số có tổng chữ số 10 tổng số lượng hoán vị {1,4,5} {2,3,5} Vậy số số có ba chữ số thỏa yêu cầu là: 2.3! = 12 số Xét số có bốn chữ số thỏa yêu cầu bất kì, ta có tổng chữ số số lốn hon l + + + 4= 10 Suy số lượng số có bốn chữ số thỏa yêu cầu số lượng hoán vị {1,2,3,4} bằng: 4.3.2.1 = 24 VÌ      15 �10 nên khơng tồn số có năm chữ số thuộc M thỏa yêu 12  24  0.12 cầu Vậy xác suất để chọn số có tổng chữ số 10: 300 Bài 28 Tại bốn đỉnh tứ diện ABCD có ghi tương ứng bốn số a, b, c, d không đồng thời nhau.Thực phép biến đổi số đỉnh tứ diện sau: Mỗi lần biến đổi ta xóa số cũ (x;y;z; t) thay vào bốn số mối {x + y + z- 3t; y + z + t - 3x; z + t + x - 3y, t + x + y - 3z) theo thứ tự Chứng minh kể từ sau lần biến đổi đầu tiên, bốn đỉnh tứ điện có đỉnh ghi số dương sau số lần thực phép biến đổi ln có đỉnh tứ diện ghi số khơng nhổ 2016 (Thanh Hóa) Lời giải   Để ý (x + y + z - 3t) + (y + z + t - 3x) + (z+ t + x - 3y) + (t + x + y - 3z) = Giả sử đến lúc khơng có số dương, tất số phải 0, dẫn đến bước trước tất số phải Mà ban đầu tất số không đồng thời nên bước chưa phải bước số bước Cứ vậy, suy cần vô hạn bước, vơ lí! Vậy sau phép biến đổi đầu tiên, ln tồn đỉnh ghi số dương, vế đầu toán Ta chứng minh sau bước đầu tiên, bước sau khiến cho số lớn bốn số bốn đỉnh tăng tiến dần dương vô Thật vậy, khơng tính tổng qt, giả sử sau bước thứ i �1 , ta bốn số x �y �z �t Khi sau bước i + ta số lớn x+ y + z - 3t Đe ý x + y + z + t = tồn bốn số số dương nên x > 0, t < Ta có x + y + z – 3t = -4 Giả sử 1 x  x  y  z  t  , mâu thuẫn! 1 t� x 4t � x Suy Vậy y, z  t 1 x Suy x � xi , i �1 i  Đặt số lớn sau bước i xi Ta có Do x1  nên xi tăng tiến dần vô Vậy tồn số m cho với i  m, x i  2016 Vậy vế sau tốn đúng, ta có điều cần chứng minh TÀI LIỆU THAM KHẢO   Diendantoanhoc.net,forum.mathscope.org,artofproblemsolving.com,nangkhieutoan.com Titu Andreescu, Dorin Andrica, Problems from the book ... z ba số thực dương thỏa mãn xyz �1 z �1 Tìm GTNN biểu thức x y  z3 F    y  x 3(1  xy ) (Thái Nguyên) Lời giải Trước hết xin nhắc lại bổ đề quen thuộc sau 1 1  � 2 Bổ đề Cho a,b > Khi

Ngày đăng: 03/05/2018, 11:12

Từ khóa liên quan

Tài liệu cùng người dùng

  • Đang cập nhật ...

Tài liệu liên quan